Đến nội dung

Hình ảnh

Đề thi Olympic toán sinh viên ĐH BK HN 2013


  • Please log in to reply
Chủ đề này có 18 trả lời

#1
viet 1846

viet 1846

    Gà con

  • Thành viên
  • 224 Bài viết
Đề thi Olympic toán sinh viên ĐH Bách Khoa Hà Nội

Môn: Giải tích

Câu 1: Tìm giới hạn: $\mathop {lim}\limits_{n \to \infty } \frac{{{1^5} + {2^5} + \cdots + {n^5}}}{{{n^6}}}$

Câu 2: Tìm $L = \mathop {lim}\limits_{m \to \infty } {\alpha _m}$ Với ${\alpha _m} = \mathop {Max}\limits_{x \in \left[ {0;1} \right]} \left( {x - {x^m}} \right)$

Câu 3: Cho hàm $u(x)$ dương liên tục trên $[0; \infty )$, hàm $\varphi \left( x \right)$ tăng và khả vi trên $[0;\infty )$, $\varphi \left( 0 \right)=1$

Biết rằng với mọi $x\ge 0$ ta có: $u\left( x \right) \le 1 + \int\limits_0^x {\frac{{\varphi '\left( t \right)}}{{\varphi \left( t \right)}}} u\left( t \right)dt$

Chứng minh: $u(x) \le \varphi \left( x \right)$ trên $[0;\infty )$


Câu 4: Cho $f_1(x)=4x^3-3x$ , $f_{n+1}=f_1(f_n)$

Tính: $\mathop {lim}\limits_{m \to \infty } \int\limits_{ - 1}^1 {f_n^2} \left( x \right)dx$


Câu 5: Tìm tất cả hàm $f(x)$ xác định trên $(0;\infty )$ Và khả vi 2 lần thỏa mãn :

$\left\{ \begin{array}{l} f'\left( x \right) > 0\\ f\left( {f'\left( x \right)} \right) = - f(x) \end{array} \right.$


Bài viết đã được chỉnh sửa nội dung bởi E. Galois: 31-01-2013 - 18:45


#2
viet 1846

viet 1846

    Gà con

  • Thành viên
  • 224 Bài viết
Các mod có gì sử hộ em với sữa mãi không được: :(

Đi thi làm được 3 bài. :(

Bài 1: Câu này khá đơn giản:

\[\mathop {lim}\limits_{n \to \infty } \frac{{{1^5} + {2^5} + \cdots + {n^5}}}{{{n^6}}} = \mathop {lim}\limits_{n \to \infty } \frac{1}{n}{\sum\limits_1^n {\left( {\frac{i}{n}} \right)} ^5} = \int\limits_0^1 {{x^5}} dx = \frac{1}{6}\]

Bài 2:

Xét hàm số:

$f(x)=x-x^m$ trên $[0;1]$

\[f'(x) = 1 - m{x^{m - 1}} = 0 \Leftrightarrow x = \frac{1}{{{m^{\frac{1}{{m - 1}}}}}} = {m^{\frac{1}{{1 - m}}}} \in (0;1)\,\forall m \ge 2\]

Ta có $f'$ đổi dấu từ dương sang âm khi $x$ qua ${m^{\frac{1}{{1 - m}}}}$

Vậy \[{\alpha _m} = \mathop {Max}\limits_{x \in \left[ {0;1} \right]} \left( {x - {x^m}} \right) = f\left( {{m^{\frac{1}{{1 - m}}}}} \right) = {m^{\frac{1}{{1 - m}}}}\left( {1 - \frac{1}{m}} \right)\]

Ta có:

\[\mathop {lim}\limits_{m \to \infty } {m^{\frac{1}{{1 - m}}}} = \mathop {lim}\limits_{m \to \infty } {e^{\frac{{\ln m}}{{1 - m}}}}{\mkern 1mu} {\mkern 1mu} = {e^{\mathop {lim}\limits_{m \to \infty } \frac{{\ln m}}{{1 - m}}}} = {e^{\mathop {lim}\limits_{m \to \infty } \frac{{ - 1}}{m}}} = 1\]

Nên \[\mathop {lim}\limits_{m \to \infty } {\alpha _m} = 1\]

Bài 4: Đặt $x=cost$ suy ra $f_1(x)=cos3t$.......$f_n(x)=cos(3^n.t)$

Ta có: $dx=-sintdt$

$x=-1 \to t=\pi$

$x=1 \to t=0$

Ta có

\[{I_n} = \int\limits_{ - 1}^1 {f_n^2} \left( x \right)dx = \int\limits_0^\pi {\sin t.{{\left[ {cos\left( {{3^n}t} \right)} \right]}^2}} dt = \frac{1}{2}\int\limits_0^\pi {\sin t.\left[ {1 + cos\left( {{{2.3}^n}t} \right)} \right]} dt\]


\[{I_n} = \frac{1}{2}\int\limits_0^\pi {\sin tdt} + \frac{1}{4}\int\limits_0^\pi {\left[ {\sin \left( {1 + {{2.3}^n}} \right)t + \sin \left( {1 - {{2.3}^n}} \right)t} \right]} dt\]


\[{I_n} = 1 - \frac{1}{4}\left[ {\frac{{cos\left( {1 + {{2.3}^n}} \right)\pi - 1}}{{1 + {{2.3}^n}}} + \frac{{cos\left( {1 - {{2.3}^n}} \right)\pi - 1}}{{1 - {{2.3}^n}}}} \right]\]

Suy ra:

\[\mathop {lim}\limits_{n \to \infty } {I_n} = 1\]

--------------------------------------------------------------------------------------------------------

hic, lúc thi thay cận là $0\to 1$ mất rồi điên mất

Bài viết đã được chỉnh sửa nội dung bởi E. Galois: 26-01-2013 - 12:57


#3
vo van duc

vo van duc

    Thiếu úy

  • ĐHV Toán Cao cấp
  • 582 Bài viết
Có đề Đại số không anh em? hi

Võ Văn Đức 17.gif       6.gif

 

 

 

 

 


#4
viet 1846

viet 1846

    Gà con

  • Thành viên
  • 224 Bài viết
Em không thi đại số anh ak. :D

#5
dangnamneu

dangnamneu

    Hạ sĩ

  • Thành viên
  • 68 Bài viết
Nhận xét: Đề thi cũng khá đơn giản, không khó nhiều câu quen thuộc.
Mình xin được góp vui Câu 3:
Thực chất đây chính là dạng của bất đẳng thức Gronwall. Ta có chứng minh như sau:
Theo điều kiện bài toán ta có:

$\frac{{u(x)}}{{1 + \int\limits_0^x {\frac{{\varphi '(t)}}{{\varphi (t)}}u(t)dt} }} \le 1,\forall x \in \left[ {0, + \infty } \right)$.

Suy ra

$\left[ {\ln \left( {1 + \int\limits_0^x {\frac{{\varphi '(t)}}{{\varphi (t)}}u(t)dt} } \right)} \right]' \le \frac{{\varphi '(x)}}{{\varphi (x)}} = \left[ {\ln \left( {\varphi (x)} \right)} \right]'$.


Lấy tích phân hai vế trên $\left[ {0,x} \right]$ của bất đẳng thức trên ta được:


$\int\limits_0^x {\left[ {\ln \left( {1 + \int\limits_0^x {\frac{{\varphi '(t)}}{{\varphi (t)}}u(t)dt} } \right)} \right]'dx \le \int\limits_0^x {\left[ {\ln \left( {\varphi (x)} \right)} \right]'dx} } \Rightarrow \ln \left( {1 + \int\limits_0^x {\frac{{\varphi '(t)}}{{\varphi (t)}}u(t)dt} } \right) \le \ln \left( {\varphi (x)} \right)$.


Suy ra
$1 + \int\limits_0^x {\frac{{\varphi '(t)}}{{\varphi (t)}}u(t)dt} \le \varphi (x) \Rightarrow u(x) \le 1 + \int\limits_0^x {\frac{{\varphi '(t)}}{{\varphi (t)}}u(t)dt} \le \varphi (x)$.


Bài toán được chứng minh.

Giáo viên môn Toán tại website : http://vted.vn


#6
dangnamneu

dangnamneu

    Hạ sĩ

  • Thành viên
  • 68 Bài viết

Bài 4: Đặt $x=cost$ suy ra $f_1(x)=cos3t$.......$f_n(x)=cos(3^n.t)$

Ta có: $dx=-sintdt$

$x=-1 \to t=\pi$

$x=1 \to t=0$

Ta có

\[{I_n} = \int\limits_{ - 1}^1 {f_n^2} \left( x \right)dx = \int\limits_0^\pi {\sin t.{{\left[ {cos\left( {{3^n}t} \right)} \right]}^2}} dt = \frac{1}{2}\int\limits_0^\pi {\sin t.\left[ {1 + cos\left( {{{2.3}^n}t} \right)} \right]} dt\]


\[{I_n} = \frac{1}{2}\int\limits_0^\pi {\sin tdt} + \frac{1}{4}\int\limits_0^\pi {\left[ {\sin \left( {1 + {{2.3}^n}} \right)t + \sin \left( {1 - {{2.3}^n}} \right)t} \right]} dt\]


\[{I_n} = 1 - \frac{1}{4}\left[ {\frac{{cos\left( {1 + {{2.3}^n}} \right)\pi - 1}}{{1 + {{2.3}^n}}} + \frac{{cos\left( {1 - {{2.3}^n}} \right)\pi - 1}}{{1 - {{2.3}^n}}}} \right]\]

Suy ra:

\[\mathop {lim}\limits_{n \to \infty } {I_n} = 1\]

--------------------------------------------------------------------------------------------------------

hic, lúc thi thay cận là $0\to 1$ mất rồi điên mất


Chỉ đơn giản như sau bạn à:


$\int\limits_{ - 1}^1 {f_n^2(t)dt} = \int\limits_{ - 1}^1 {c{\rm{o}}{{\rm{s}}^2}\left( {{3^n}t} \right)dt} = 1 + \frac{1}{2}\int\limits_{ - 1}^1 {c{\rm{os}}\left( {{{2.3}^n}t} \right)dt} = 1 + \frac{1}{{{{2.3}^n}}}\sin \left( {{{2.3}^n}} \right) \to 1$


Bài viết đã được chỉnh sửa nội dung bởi phudinhgioihan: 27-01-2013 - 15:09

Giáo viên môn Toán tại website : http://vted.vn


#7
viet 1846

viet 1846

    Gà con

  • Thành viên
  • 224 Bài viết

Chỉ đơn giản như sau bạn à:


$\int\limits_{ - 1}^1 {f_n^2(t)dt} = \int\limits_{ - 1}^1 {c{\rm{o}}{{\rm{s}}^2}\left( {{3^n}t} \right)dt} = 1 + \frac{1}{2}\int\limits_{ - 1}^1 {c{\rm{os}}\left( {{{2.3}^n}t} \right)dt} = 1 + \frac{1}{{{{2.3}^n}}}\sin \left( {{{2.3}^n}} \right) \to 1$


Mình nghĩ là bài bạn có vấn đề, việc thay $x=cost$ thì ta cũng cần phải đổi vi phân chứ.

#8
ssupermeo

ssupermeo

    Binh nhì

  • Thành viên
  • 13 Bài viết

Bài 1. cho MT $A=\begin{pmatrix} -1 &3 \\ -1&2 \end{pmatrix}$ và $B=\begin{pmatrix} \frac{\sqrt{3}-1}{2} &-1 \\ \frac{1}{2}& \frac{\sqrt{3}+1}{2} \end{pmatrix}$

a. Cmr $A^2-A+E=0$ tính $f(A)=E+\sum_{2013}^{k=1}(-1)^kA^k$ với E là ma trận đơn vị cấp 2

b. Tính $B^{2016}$



Bài 2. Cho ma trận $A$ là một ma trận thực, vuông cấp $n$. CMR $det(A-A^t)\geq 0$ với $A^t$ là ma trận chuyển vị của ma trận A



Bài 3. Cho ma trận A là ma trận vuông cấp n. Vết của $A$, kí hiệu $tr(A)$ là tổng các phần tử chéo của $A$. Ma trận $A$ gọi là ma trận lũy đẳng nếu $A^2=A$. CMR:

a. Nếu $A$ là ma trận lũy đẳng thì $A$ chéo hóa được

b. $A$ là ma trận lũy đẳng khi và chỉ khi $rank(A)=tr(A)$ và $rank(E-A)= tr(E-A)$



Bài 4. Tính định thức của ma trận vuông cấp 2013 $A=[a_{ij}]$ với:
$a_{ij}=\left\{\begin{matrix} b & khi & i< j\\ a & khi &i=j \\ -b & khi & i> j \end{matrix}\right.$



Bài 5. Cho đa thức $f(x)\in R[x]$ có ít nhất 2 nghiệm thực.


CMR đa thức $p(x)=f(x)-4026f'(x)+2013f''(x)$ cũng có ít nhất 2 nghiệm


Bài viết đã được chỉnh sửa nội dung bởi phudinhgioihan: 31-01-2013 - 10:26


#9
vo van duc

vo van duc

    Thiếu úy

  • ĐHV Toán Cao cấp
  • 582 Bài viết
Câu 1.a)

Dùng định lý Caley-Haminton hay tính toán trực tiếp ta có đẳng thức $A^{2}-A+E=O$

Ta có:

$f(A)=E-A+A^{2}- A^{3} + A^{4}- A^{5}+...+ A^{2012}- A^{2013}$

$=(E-A)+(E-A). A^{2}+ (E-A). A^{4}+...+ (E-A). A^{2012}$

$=(E-A).( E+A^{2}+ A^{4}+...+ A^{2012})$

Ta dễ dàng chứng minh được rằng

$A^{6k}=E$

$A^{6k+1}=A$

$A^{6k+2}=A-E$

$A^{6k+3}=-E$

$A^{6k+4}=-A$

$A^{6k+5}=E-A$

Suy ra:

$A^{2}+ A^{4}+ A^{6}=O$

$A^{8}+ A^{10}+ A^{12}=O$

...

$A^{2008}+ A^{2010}+ A^{2012}=O$

Vậy $f(A)=E-A$

..............
Online bằng di động nên không định dạng cho đẹp được. Anh em thông cảm nha!

Bài viết đã được chỉnh sửa nội dung bởi vo van duc: 30-01-2013 - 15:44

Võ Văn Đức 17.gif       6.gif

 

 

 

 

 


#10
vo van duc

vo van duc

    Thiếu úy

  • ĐHV Toán Cao cấp
  • 582 Bài viết
Câu 1.b)

Ta phân tích

$A=\begin{pmatrix} \frac{\sqrt{3}-1}{2} & -1 \\ \frac{1}{2} & \frac{\sqrt{3}+1}{2} \end{pmatrix}$

$=\begin{pmatrix} \cos \frac{\pi}{6}-\sin \frac{\pi}{6} & -2 \sin \frac{\pi}{6} \\ \sin \frac{\pi}{6} & \cos \frac{\pi}{6}+\sin \frac{\pi}{6} \end{pmatrix}$

Chứng minh quy nạp ta có

$A^{n}=\begin{pmatrix} \cos n\frac{\pi}{6}-\sin n\frac{\pi}{6} & -2 \sin n\frac{\pi}{6} \\ \sin n\frac{\pi}{6} & \cos n\frac{\pi}{6}+\sin n\frac{\pi}{6} \end{pmatrix}$

Với $n=2016$ thì $A^{2016}=\begin{pmatrix} 1 & 0 \\ 0 & 1 \end{pmatrix}$

Võ Văn Đức 17.gif       6.gif

 

 

 

 

 


#11
vo van duc

vo van duc

    Thiếu úy

  • ĐHV Toán Cao cấp
  • 582 Bài viết
Câu 4:

Xét định thức cấp n

$D_{n}=\begin{vmatrix} -b & -b & -b & \cdots & -b & -b \\ a & -b & -b & \cdots & -b & -b \\ b & a & -b & \cdots & -b & -b \\ \vdots & \vdots & \vdots & \ddots & \vdots & \vdots \\ b & b & b & \cdots & -b & -b \\ b & b & b & \cdots & a & b \end{vmatrix}$

Thay cột 1 bằng cột 1 cộng cột n, ta có

$D_{n}=\begin{vmatrix} a-b & -b & -b & \cdots & -b & -b \\ 0 & -b & -b & \cdots & -b & -b \\ 0 & a & -b & \cdots & -b & -b \\ \vdots & \vdots & \vdots & \ddots & \vdots & \vdots \\ 0 & b & b & \cdots & -b & -b \\ a+b & b & b & \cdots & a & b \end{vmatrix}$

$=(a-b).D_{n-1} + (a+b).(-1)^{n+1}.D$

Với $D=\begin{vmatrix} -b & -b & -b & \cdots & -b & -b \\ a & -b & -b & \cdots & -b & -b \\ b & a & -b & \cdots & -b & -b \\ \vdots & \vdots & \vdots & \ddots & \vdots & \vdots \\ b & b & b & \cdots & -b & -b \\ b & b & b & \cdots & a & -b \end{vmatrix}_{n-1}$

$=\begin{vmatrix} -b & -b & -b & \cdots & -b & -b \\ a+b & 0 & 0 & \cdots & 0 & 0 \\ 2b & a+b & 0 & \cdots & 0 & 0 \\ \vdots & \vdots & \vdots & \ddots & \vdots & \vdots \\ 2b & 2b & 2b & \cdots & 0 & 0 \\ 2b & 2b & 2b & \cdots & a+b & 0 \end{vmatrix}_{n-1}$

$=(-b).(-1)^{n}.(a+b)^{n-2}$

Vậy $D_{n}=(a-b).D_{n-1}+b.(a+b)^{n-1}$

Ta có:

$D_{1}=a=\frac{1}{2}.\left [ (a-b)^{1} + (a+b)^{1} \right ]$

$D_{2}=a^{2}+b^{2}=\frac{1}{2}.\left [ (a-b)^{2} + (a+b)^{2} \right ]$

Dự đoán $D_{n}=\frac{1}{2}.\left [ (a-b)^{n} + (a+b)^{n} \right ] (*)$

Thật vậy. Ta chứng minh $(*)$ bằng quy nạp.

Giả sử $(*)$ đúng với $n=k$, tức là $D_{k}=\frac{1}{2}.\left [ (a-b)^{k} + (a+b)^{k} \right ]$

Ta có

$D_{k+1}=(a-b).D_{k}+b.(a+b)^{k}$

$=(a-b).\frac{1}{2}.\left [ (a-b)^{k}+(a+b)^{k} \right ] +b.(a+b)^{k}$

$=\frac{1}{2}.(a-b)^{k+1}+ \frac{1}{2}.(a-b).(a+b)^{k}+ b.(a+b)^{k}$

$=\frac{1}{2}.(a-b)^{k+1}+\left [ \frac{1}{2}.(a-b)+b \right ].(a+b)^{k}$

$=\frac{1}{2}.\left [ (a-b)^{k+1} + (a+b)^{k+1} \right ]$

Vậy $(*)$ đúnh với $n=k+1$.
Theo nguyên lý quy nạp thì ta chứng minh được

$D_{n}=\frac{1}{2}.\left [ (a-b)^{n} + (a+b)^{n} \right ]$


Thay $n=2013$ ta có đáp số của đề.

..........................
Soạn trên dđ nên chắc có sai sót. phudinhgioihan sửa lại giúp anh với nha! hi

Bài viết đã được chỉnh sửa nội dung bởi vo van duc: 30-01-2013 - 09:52

Võ Văn Đức 17.gif       6.gif

 

 

 

 

 


#12
vo van duc

vo van duc

    Thiếu úy

  • ĐHV Toán Cao cấp
  • 582 Bài viết
Câu 2:

Nhận xét rằng ma trận $A-A^{t}$ là ma trận phản đối xứng. Đang nghiên cứu về các tính chất về định thức, giá trị riêng của ma trận phản đối xứng để giải quyết bài này. Chỉ là ý tưởng thôi.

Anh em giúp đở một số tính chất của ma trận phản đối xứng đi. Một ma trận đẹp. hi

Võ Văn Đức 17.gif       6.gif

 

 

 

 

 


#13
GreatLuke

GreatLuke

    Binh nhất

  • Thành viên
  • 46 Bài viết
Câu 2 Đại số:

Ma trận phản đối xứng $M$ nếu có 1 giá trị riêng là $x$ thì cũng có giá trị riêng là $-x$.

Nếu nó có bậc lẻ thì phải có ít nhất 1 giá trị riêng bằng $0$ nên có $detM=0$.

Nếu bậc của ma trận là 1 số chẵn:

Chứng minh (bằng phản chứng) ma trận phản đối xứng $M$ nếu có 1 giá trị riêng x khác $0$ thì đó là số thuần ảo.

Mặt khác $-x$ cũng là 1 giá trị riêng của M.

$detM$ bằng tích các giá trị riêng nên đi đến kết luận $detM \geq 0$.

Bài viết đã được chỉnh sửa nội dung bởi GreatLuke: 30-01-2013 - 23:38


#14
GreatLuke

GreatLuke

    Binh nhất

  • Thành viên
  • 46 Bài viết

Câu 1.a)

Dùng định lý Caley-Haminton hay tính toán trực tiếp ta có đẳng thức $A^{2}-A+E=O$

Ta có:

$f(A)=E-A+A^{2}- A^{3} + A^{4}- A^{5}+...+ A^{2}- A^{2}$

$=(E-A)+(E-A). A^{2}+ (E-A). A^{4}+...+ (E-A). A^{2012}$

$=(E-A).( E+A^{2}+ A^{4}+...+ A^{2012})$

Ta dễ dàng chứng minh được rằng

$A^{6k}=E$

$A^{6k+1}=A$

$A^{6k+2}=A-E$

$A^{6k+3}=-E$

$A^{6k+4}=-A$

$A^{6k+5}=E-A$

Suy ra:

$A^{2}+ A^{4}+ A^{6}=O$

$A^{8}+ A^{10}+ A^{12}=O$

...

$A^{2008}+ A^{2010}+ A^{2012}=O$

Vậy $f(A)=E-A$

..............
Online bằng di động nên không định dạng cho đẹp được. Anh em thông cảm nha!

Câu này em tính ra là $f(A)=E$ cơ. mà nhóm cũng đơn giản chứ không dài như anh

#15
GreatLuke

GreatLuke

    Binh nhất

  • Thành viên
  • 46 Bài viết

3. Cho ma trận A là ma trận vuông cấp n. Vết của A, kí hiệu tr(A) là tổng các phần tử chéo của A. Ma trận A gọi là ma trận lũy đẳng nếu $A^2=A$. CMR:

a. Nếu A là ma trận lũy đẳng thì A chéo hóa được

b. A là ma trận lũy đẳng khi và chỉ khi $rank(A)=tr(A)$ và $rank(E-A)= tr(E-A)$

a. Đa thức triệt tiêu của $A$ tách đơn nên $A$ chéo hóa được.

b.
Thuận:

$A$ lũy đẳng nên $A$ chéo hóa được. Các giá trị riêng của $A$ là $0$ hoặc $1$

Khi đó $rankA=n-dim(Ker(A))=n-dim(KGCR(A,0))=dim(KGCR(A,1))=tr(A)$

Dễ dàng chứng minh được $rank(E-A)=tr(E-A)$(có thể sử dụng dạng chéo hóa của ma trận $A$).

Đảo:

Vì $tr(A)=rank(A)$ và $tr(E-A)=rank(E-A)$ nên $rank(A)+rank(E-A)=tr(A)+tr(E-A)=n$.

Chứng minh đẳng thức sau: $rank(A)+rank(E-A)=n+rank((E-A)A)$.
Trước hết có bất đẳng thức $rank(A)+rank(B)\leq n+rank(AB)$.

Mặt khác ta có:
Nếu $X_{1}$ thuộc $Ker(A)$, $X_{2}$ thuộc $Ker(E-A)$ . Xét $X_{1}+X_{2}=0$.
Khi đó $A(X_{1}+X_{2})=0$ nên $X_{2}=0$(do $AX_{1}=0$;$AX_{2}=X_{2}$) và $X_{1}=0$. Vậy $Ker(A)$ và $Ker(E-A)$ có tổng trực tiếp.

Với mọi $Y$ thuộc $Ker(A)+Ker(E-A)$ thì $Y$ thuộc $Ker(A(E-A))$. Vậy $dim(Ker(A)+Ker(E-A))=dim(Ker(A))+dim(Ker(E-A))\leq dim(Ker(A(E-A))$. Vì $dim(Ker(A))+rank(A)=n$ nên $rank(A)+rank(E-A)\geq n+rank(A(E-A))$.

Kết hợp lại ta có $rank(A)+rank(E-A)=n+rank((E-A)A)=n$.

Vậy $rank(A-A^{2})=0$ hay $A$ lũy đẳng

p/s:Mình tự thấy cách của mình ko hay. Bạn nào có cách khác thì post lên cho mình học tập.

Bài viết đã được chỉnh sửa nội dung bởi GreatLuke: 30-01-2013 - 17:50


#16
phudinhgioihan

phudinhgioihan

    PĐGH$\Leftrightarrow$TDST

  • Biên tập viên
  • 348 Bài viết


Bài 5. Cho đa thức $f(x)\in R[x]$ có ít nhất 2 nghiệm thực.


CMR đa thức $p(x)=f(x)-4026f'(x)+2013f''(x)$ cũng có ít nhất 2 nghiệm


Nếu $f$ bậc chẵn thì 2 bài tổng quát mình đã giải ở đây:

http://diendantoanho...-p/#entry249

Nếu đa thức $f$ bậc lẻ ( $\deg (f)=2n+1 \;, n \in \mathbb{N}$), khi đó, tồn tại $2 $ số thực $a_i$ sao cho

$$f(x)=(x-a_1)(x-a_2) H_{2n-1}(x)$$

Với $H_{2n-1}$ là đa thức bậc $2n-1$ và do đó bậc lẻ nên có ít nhất một nghiệm thực, do đó $f$ có ít nhất $3$ nghiệm thực (kể cả bội).


Với $t_1 \neq 0$

Xét $q(x)=e^{\frac{x}{t_1}} f(x)$

Do $f$ có ít nhất 3 nghiệm thực nên $q(x)$ cũng có ít nhất 3 nghiệm thực.

$$q'(x)=e^{\frac{x}{t_1}} \dfrac{1}{t_1}(f(x)+t_1f'(x))$$

suy ra $q'(x)=0$ có ít nhất 2 nghiệm thực, do đó $r(x)=f(x)+t_1f'(x)$ có ít nhất 2 nghiệm thực. Do $\deg( r )=\deg(f)$ nên $r$ có ít nhất 3 nghiệm thực.

Tương tự, với $t_2 \neq 0$ ta cũng có $h(x)=r(x)+t_2r'(x)=f(x)+(t_1+t_2)f'(x)+t_1t_2f''(x)$ có ít nhất 3 nghiệm thực.

Chọn $t_1+t_2=-4026 \;, t_1t_2=2013$, do phương trình $x^2-4026x+2013=0$ có 2 nghiệm thực nên tồn tại cắp số thực $t_1,t_2$ như thế. Vây ta có

$p(x)=f(x)-4026f'(x)+2013f''(x)$ có ít nhất 3 nghiệm thực.

Phủ định của giới hạn Hình đã gửi

Đó duy sáng tạo ! Hình đã gửi


https://phudinhgioihan.wordpress.com/

#17
LangTu Mua Bui

LangTu Mua Bui

    Binh nhất

  • Thành viên
  • 43 Bài viết
Câu 5 
$\left\{ \begin{array}{l} f'\left( x \right) > 0\\ f\left( {f'\left( x \right)} \right) = - f(x) \end{array} \right.$

Ta có $f(f'(x))=-f(x)$

Thay $ x=f'(x) \Rightarrow f(f'(f'(x))))=f(x)$ Do $f'(x)>0 \Rightarrow f'(f'(x))=x  (1)$

$ \Rightarrow  f(f'(x))=-f(x)$

Do hàm khả vi cấp 2 nên Ta  đạo hàm 2 vế theo x $ \Rightarrow f''(x)f'(f'(x))=-f'(x)(2)$
 
Từ $ (1)(2)f''(x).f'(f'(x))\Leftrightarrow f''(x)x+f'(x)=0$

$ \Rightarrow f(x)=a\ln{x}+b$ Thay ngược lại đề bài $\Rightarrow  a=b=1 \Rightarrow f(x)=\ln{x} $

Bài viết đã được chỉnh sửa nội dung bởi LangTu Mua Bui: 05-12-2015 - 12:03


#18
LangTu Mua Bui

LangTu Mua Bui

    Binh nhất

  • Thành viên
  • 43 Bài viết
 Cách khác cho câu 3

$u(x)\leq 1+\int_{0}^{x}\frac{\varphi '(t)u(t)dt}{\varphi (t)}$

Dễ thấy $ u(0) \leq 1$
 
$\Leftrightarrow u(x)-\varphi (x)  \leq  \int_{0}^{x} \frac{\varphi '(t)u(t)dt}{\varphi (t)}-\int_{0}^{x}\varphi '(t)dt=\int_{0}^{x}\left (\varphi '(t)( \frac{u(t)-\varphi (t)}{\varphi (t)})  \right ) dt $

Do $\varphi (t)$ đồng biến và $\varphi(0)=1 \Rightarrow \varphi (t)\geq 1 \forall t\in [0;\infty) $

$\Rightarrow u(x)-\varphi (x) \leq  \int_{0}^{x}\left (\varphi '(t)(u(x)-1)  \right ) dt=u(x)-\varphi (x)-\int_{0}^{x}\varphi '(x) $

$\Rightarrow \varphi(x)<1-\int_{0}^{x}u'(t)\varphi (t)dt ;\varphi(x)\geq 1 \forall x\in [0;\infty] \Rightarrow  u'(t)<0 $

Xét hàm số $g(x)=u(x)-\varphi (x) $

$g'(x)=u'(x)-\varphi' (x) <0 ;g(0)=u(0)-\varphi (0)<0 \Rightarrow g(x)<0 \forall x\in [0;\infty]$


#19
luuvanthai

luuvanthai

    Sĩ quan

  • Thành viên
  • 373 Bài viết

 

 Cách khác cho câu 3

$u(x)\leq 1+\int_{0}^{x}\frac{\varphi '(t)u(t)dt}{\varphi (t)}$

Dễ thấy $ u(0) \leq 1$
 
$\Leftrightarrow u(x)-\varphi (x)  \leq  \int_{0}^{x} \frac{\varphi '(t)u(t)dt}{\varphi (t)}-\int_{0}^{x}\varphi '(t)dt=\int_{0}^{x}\left (\varphi '(t)( \frac{u(t)-\varphi (t)}{\varphi (t)})  \right ) dt $

Do $\varphi (t)$ đồng biến và $\varphi(0)=1 \Rightarrow \varphi (t)\geq 1 \forall t\in [0;\infty) $

$\Rightarrow u(x)-\varphi (x) \leq  \int_{0}^{x}\left (\varphi '(t)(u(x)-1)  \right ) dt=u(x)-\varphi (x)-\int_{0}^{x}\varphi '(x) $

$\Rightarrow \varphi(x)<1-\int_{0}^{x}u'(t)\varphi (t)dt ;\varphi(x)\geq 1 \forall x\in [0;\infty] \Rightarrow  u'(t)<0 $

Xét hàm số $g(x)=u(x)-\varphi (x) $

$g'(x)=u'(x)-\varphi' (x) <0 ;g(0)=u(0)-\varphi (0)<0 \Rightarrow g(x)<0 \forall x\in [0;\infty]$

 

Dòng này có vấn đề??






2 người đang xem chủ đề

0 thành viên, 2 khách, 0 thành viên ẩn danh